Asymptotic behaviour of the sequence of the number of groups of order $n$











up vote
0
down vote

favorite












Let $f(n) = text{# of groups of order} n$. I want to study the asymptotic behaviour of this sequence as $n to infty$. Clearly $lim inf f(n) = 1$ and $lim sup f(n) = infty$, so the sequence is jumping around.



I'm wondering if there exists a "nice" function $g$ such that $$0<limsup_{n to +infty} frac{f(n)}{g(n)}<+infty$$










share|cite|improve this question






















  • I think such precise knowledge of the asymptotics of $f(n)$ is an open question.
    – verret
    Nov 16 at 19:01















up vote
0
down vote

favorite












Let $f(n) = text{# of groups of order} n$. I want to study the asymptotic behaviour of this sequence as $n to infty$. Clearly $lim inf f(n) = 1$ and $lim sup f(n) = infty$, so the sequence is jumping around.



I'm wondering if there exists a "nice" function $g$ such that $$0<limsup_{n to +infty} frac{f(n)}{g(n)}<+infty$$










share|cite|improve this question






















  • I think such precise knowledge of the asymptotics of $f(n)$ is an open question.
    – verret
    Nov 16 at 19:01













up vote
0
down vote

favorite









up vote
0
down vote

favorite











Let $f(n) = text{# of groups of order} n$. I want to study the asymptotic behaviour of this sequence as $n to infty$. Clearly $lim inf f(n) = 1$ and $lim sup f(n) = infty$, so the sequence is jumping around.



I'm wondering if there exists a "nice" function $g$ such that $$0<limsup_{n to +infty} frac{f(n)}{g(n)}<+infty$$










share|cite|improve this question













Let $f(n) = text{# of groups of order} n$. I want to study the asymptotic behaviour of this sequence as $n to infty$. Clearly $lim inf f(n) = 1$ and $lim sup f(n) = infty$, so the sequence is jumping around.



I'm wondering if there exists a "nice" function $g$ such that $$0<limsup_{n to +infty} frac{f(n)}{g(n)}<+infty$$







group-theory






share|cite|improve this question













share|cite|improve this question











share|cite|improve this question




share|cite|improve this question










asked Nov 16 at 0:21









MathematicsStudent1122

7,31422160




7,31422160












  • I think such precise knowledge of the asymptotics of $f(n)$ is an open question.
    – verret
    Nov 16 at 19:01


















  • I think such precise knowledge of the asymptotics of $f(n)$ is an open question.
    – verret
    Nov 16 at 19:01
















I think such precise knowledge of the asymptotics of $f(n)$ is an open question.
– verret
Nov 16 at 19:01




I think such precise knowledge of the asymptotics of $f(n)$ is an open question.
– verret
Nov 16 at 19:01










1 Answer
1






active

oldest

votes

















up vote
0
down vote













It may really depend on the prime factorization of $n$. For example, you can see here that there's a large gap between the number of groups of order $2^{k} - 1$ and $2^{k}$.
However, there's an asymptotic formula for the number of groups of order $p^{n}$ for a fixed prime $p$:
$$
f(n, p) = p^{(2/27 + o(1))n^{3}}.
$$

See Higman-Sims asymptotic formula.






share|cite|improve this answer





















    Your Answer





    StackExchange.ifUsing("editor", function () {
    return StackExchange.using("mathjaxEditing", function () {
    StackExchange.MarkdownEditor.creationCallbacks.add(function (editor, postfix) {
    StackExchange.mathjaxEditing.prepareWmdForMathJax(editor, postfix, [["$", "$"], ["\\(","\\)"]]);
    });
    });
    }, "mathjax-editing");

    StackExchange.ready(function() {
    var channelOptions = {
    tags: "".split(" "),
    id: "69"
    };
    initTagRenderer("".split(" "), "".split(" "), channelOptions);

    StackExchange.using("externalEditor", function() {
    // Have to fire editor after snippets, if snippets enabled
    if (StackExchange.settings.snippets.snippetsEnabled) {
    StackExchange.using("snippets", function() {
    createEditor();
    });
    }
    else {
    createEditor();
    }
    });

    function createEditor() {
    StackExchange.prepareEditor({
    heartbeatType: 'answer',
    convertImagesToLinks: true,
    noModals: true,
    showLowRepImageUploadWarning: true,
    reputationToPostImages: 10,
    bindNavPrevention: true,
    postfix: "",
    imageUploader: {
    brandingHtml: "Powered by u003ca class="icon-imgur-white" href="https://imgur.com/"u003eu003c/au003e",
    contentPolicyHtml: "User contributions licensed under u003ca href="https://creativecommons.org/licenses/by-sa/3.0/"u003ecc by-sa 3.0 with attribution requiredu003c/au003e u003ca href="https://stackoverflow.com/legal/content-policy"u003e(content policy)u003c/au003e",
    allowUrls: true
    },
    noCode: true, onDemand: true,
    discardSelector: ".discard-answer"
    ,immediatelyShowMarkdownHelp:true
    });


    }
    });














     

    draft saved


    draft discarded


















    StackExchange.ready(
    function () {
    StackExchange.openid.initPostLogin('.new-post-login', 'https%3a%2f%2fmath.stackexchange.com%2fquestions%2f3000534%2fasymptotic-behaviour-of-the-sequence-of-the-number-of-groups-of-order-n%23new-answer', 'question_page');
    }
    );

    Post as a guest















    Required, but never shown

























    1 Answer
    1






    active

    oldest

    votes








    1 Answer
    1






    active

    oldest

    votes









    active

    oldest

    votes






    active

    oldest

    votes








    up vote
    0
    down vote













    It may really depend on the prime factorization of $n$. For example, you can see here that there's a large gap between the number of groups of order $2^{k} - 1$ and $2^{k}$.
    However, there's an asymptotic formula for the number of groups of order $p^{n}$ for a fixed prime $p$:
    $$
    f(n, p) = p^{(2/27 + o(1))n^{3}}.
    $$

    See Higman-Sims asymptotic formula.






    share|cite|improve this answer

























      up vote
      0
      down vote













      It may really depend on the prime factorization of $n$. For example, you can see here that there's a large gap between the number of groups of order $2^{k} - 1$ and $2^{k}$.
      However, there's an asymptotic formula for the number of groups of order $p^{n}$ for a fixed prime $p$:
      $$
      f(n, p) = p^{(2/27 + o(1))n^{3}}.
      $$

      See Higman-Sims asymptotic formula.






      share|cite|improve this answer























        up vote
        0
        down vote










        up vote
        0
        down vote









        It may really depend on the prime factorization of $n$. For example, you can see here that there's a large gap between the number of groups of order $2^{k} - 1$ and $2^{k}$.
        However, there's an asymptotic formula for the number of groups of order $p^{n}$ for a fixed prime $p$:
        $$
        f(n, p) = p^{(2/27 + o(1))n^{3}}.
        $$

        See Higman-Sims asymptotic formula.






        share|cite|improve this answer












        It may really depend on the prime factorization of $n$. For example, you can see here that there's a large gap between the number of groups of order $2^{k} - 1$ and $2^{k}$.
        However, there's an asymptotic formula for the number of groups of order $p^{n}$ for a fixed prime $p$:
        $$
        f(n, p) = p^{(2/27 + o(1))n^{3}}.
        $$

        See Higman-Sims asymptotic formula.







        share|cite|improve this answer












        share|cite|improve this answer



        share|cite|improve this answer










        answered Nov 16 at 0:32









        Seewoo Lee

        5,941826




        5,941826






























             

            draft saved


            draft discarded



















































             


            draft saved


            draft discarded














            StackExchange.ready(
            function () {
            StackExchange.openid.initPostLogin('.new-post-login', 'https%3a%2f%2fmath.stackexchange.com%2fquestions%2f3000534%2fasymptotic-behaviour-of-the-sequence-of-the-number-of-groups-of-order-n%23new-answer', 'question_page');
            }
            );

            Post as a guest















            Required, but never shown





















































            Required, but never shown














            Required, but never shown












            Required, but never shown







            Required, but never shown

































            Required, but never shown














            Required, but never shown












            Required, but never shown







            Required, but never shown







            Popular posts from this blog

            How do I know what Microsoft account the skydrive app is syncing to?

            When does type information flow backwards in C++?

            Grease: Live!